How to Write the AP Lang Rhetorical Analysis Essay (With Example)

November 27, 2023

how to write AP Lang rhetorical analysis essay example

Feeling intimidated by the AP Lang Rhetorical Analysis Essay? We’re here to help demystify. Whether you’re cramming for the AP Lang exam right now or planning to take the test down the road, we’ve got crucial rubric information, helpful tips, and an essay example to prepare you for the big day. This post will cover 1) What is the AP Lang Rhetorical Analysis Essay? 2) AP Lang Rhetorical Analysis Rubric 3) AP Lang Rhetorical Analysis: Sample Prompt 4) AP Lang Rhetorical Analysis Essay Example 5)AP Lang Rhetorical Analysis Essay Example: Why It Works

What is the AP Lang Rhetorical Analysis Essay?

The AP Lang Rhetorical Analysis Essay is one of three essays included in the written portion of the AP English Exam. The full AP English Exam is 3 hours and 15 minutes long, with the first 60 minutes dedicated to multiple-choice questions. Once you complete the multiple-choice section, you move on to three equally weighted essays that ask you to synthesize, analyze, and interpret texts and develop well-reasoned arguments. The three essays include:

Synthesis essay: You’ll review various pieces of evidence and then write an essay that synthesizes (aka combines and interprets) the evidence and presents a clear argument. Read our write up on How to Write the AP Lang Synthesis Essay here.

Argumentative essay: You’ll take a stance on a specific topic and argue your case.

Rhetorical essay: You’ll read a provided passage, then analyze the author’s rhetorical choices and develop an argument that explains why the author made those rhetorical choices.

AP Lang Rhetorical Analysis Rubric

The AP Lang Rhetorical Analysis Essay is graded on just 3 rubric categories: Thesis, Evidence and Commentary, and Sophistication . At a glance, the rubric categories may seem vague, but AP exam graders are actually looking for very particular things in each category. We’ll break it down with dos and don’ts for each rubric category:

Thesis (0-1 point)

There’s nothing nebulous when it comes to grading AP Lang Rhetorical Analysis Essay thesis. You either have one or you don’t. Including a thesis gets you one point closer to a high score and leaving it out means you miss out on one crucial point. So, what makes a thesis that counts?

  • Make sure your thesis argues something about the author’s rhetorical choices. Making an argument means taking a risk and offering your own interpretation of the provided text. This is an argument that someone else might disagree with.
  • A good test to see if you have a thesis that makes an argument. In your head, add the phrase “I think that
” to the beginning of your thesis. If what follows doesn’t logically flow after that phrase (aka if what follows isn’t something you and only you think), it’s likely you’re not making an argument.
  • Avoid a thesis that merely restates the prompt.
  • Avoid a thesis that summarizes the text but does not make an argument.

Evidence and Commentary (0-4 points)

This rubric category is graded on a scale of 0-4 where 4 is the highest grade. Per the AP Lang Rhetorical Analysis rubric, to get a 4, you’ll want to:

  • Include lots of specific evidence from the text. There is no set golden number of quotes to include, but you’ll want to make sure you’re incorporating more than a couple pieces of evidence that support your argument about the author’s rhetorical choices.
  • Make sure you include more than one type of evidence, too. Let’s say you’re working on your essay and have gathered examples of alliteration to include as supporting evidence. That’s just one type of rhetorical choice, and it’s hard to make a credible argument if you’re only looking at one type of evidence. To fix that issue, reread the text again looking for patterns in word choice and syntax, meaningful figurative language and imagery, literary devices, and other rhetorical choices, looking for additional types of evidence to support your argument.
  • After you include evidence, offer your own interpretation and explain how this evidence proves the point you make in your thesis.
  • Don’t summarize or speak generally about the author and the text. Everything you write must be backed up with evidence.
  • Don’t let quotes speak for themselves. After every piece of evidence you include, make sure to explain your interpretation. Also, connect the evidence to your overarching argument.

Sophistication (0-1 point)

In this case, sophistication isn’t about how many fancy vocabulary words or how many semicolons you use. According to College Board , one point can be awarded to AP Lang Rhetorical Analysis essays that “demonstrate sophistication of thought and/or a complex understanding of the rhetorical situation” in any of these three ways:

  • Explaining the significance or relevance of the writer’s rhetorical choices.
  • Explaining the purpose or function of the passage’s complexities or tensions.
  • Employing a style that is consistently vivid and persuasive.

Note that you don’t have to achieve all three to earn your sophistication point. A good way to think of this rubric category is to consider it a bonus point that you can earn for going above and beyond in depth of analysis or by writing an especially persuasive, clear, and well-structured essay. In order to earn this point, you’ll need to first do a good job with your thesis, evidence, and commentary.

  • Focus on nailing an argumentative thesis and multiple types of evidence. Getting these fundamentals of your essay right will set you up for achieving depth of analysis.
  • Explain how each piece of evidence connects to your thesis.
  • Spend a minute outlining your essay before you begin to ensure your essay flows in a clear and cohesive way.
  • Steer clear of generalizations about the author or text.
  • Don’t include arguments you can’t prove with evidence from the text.
  • Avoid complex sentences and fancy vocabulary words unless you use them often. Long, clunky sentences with imprecisely used words are hard to follow.

AP Lang Rhetorical Analysis: Sample Prompt

The sample prompt below is published online by College Board and is a real example from the 2021 AP Exam. The prompt provides background context, essay instructions, and the text you need to analyze. For sake of space, we’ve included the text as an image you can click to read. After the prompt, we provide a sample high scoring essay and then explain why this AP Lang Rhetorical Analysis essay example works.

Suggested time—40 minutes.

(This question counts as one-third of the total essay section score.)

On February 27, 2013, while in office, former president Barack Obama delivered the following address dedicating the Rosa Parks statue in the National Statuary Hall of the United States Capitol building. Rosa Parks was an African American civil rights activist who was arrested in 1955 for refusing to give up her seat on a segregated bus in Montgomery, Alabama. Read the passage carefully. Write an essay that analyzes the rhetorical choices Obama makes to convey his message.

In your response you should do the following:

  • Respond to the prompt with a thesis that analyzes the writer’s rhetorical choices.
  • Select and use evidence to support your line of reasoning.
  • Explain how the evidence supports your line of reasoning.
  • Demonstrate an understanding of the rhetorical situation.
  • Use appropriate grammar and punctuation in communicating your argument.

AP Lang Rhetorical Analysis Essay Example

In his speech delivered in 2013 at the dedication of Rosa Park’s statue, President Barack Obama acknowledges everything that Parks’ activism made possible in the United States. Telling the story of Parks’ life and achievements, Obama highlights the fact that Parks was a regular person whose actions accomplished enormous change during the civil rights era. Through the use of diction that portrays Parks as quiet and demure, long lists that emphasize the extent of her impacts, and Biblical references, Obama suggests that all of us are capable of achieving greater good, just as Parks did.

Although it might be a surprising way to start to his dedication, Obama begins his speech by telling us who Parks was not: “Rosa Parks held no elected office. She possessed no fortune” he explains in lines 1-2. Later, when he tells the story of the bus driver who threatened to have Parks arrested when she refused to get off the bus, he explains that Parks “simply replied, ‘You may do that’” (lines 22-23). Right away, he establishes that Parks was a regular person who did not hold a seat of power. Her protest on the bus was not part of a larger plan, it was a simple response. By emphasizing that Parks was not powerful, wealthy, or loud spoken, he implies that Parks’ style of activism is an everyday practice that all of us can aspire to.

AP Lang Rhetorical Analysis Essay Example (Continued)

Even though Obama portrays Parks as a demure person whose protest came “simply” and naturally, he shows the importance of her activism through long lists of ripple effects. When Parks challenged her arrest, Obama explains, Martin Luther King, Jr. stood with her and “so did thousands of Montgomery, Alabama commuters” (lines 27-28). They began a boycott that included “teachers and laborers, clergy and domestics, through rain and cold and sweltering heat, day after day, week after week, month after month, walking miles if they had to
” (lines 28-31). In this section of the speech, Obama’s sentences grow longer and he uses lists to show that Parks’ small action impacted and inspired many others to fight for change. Further, listing out how many days, weeks, and months the boycott lasted shows how Parks’ single act of protest sparked a much longer push for change.

To further illustrate Parks’ impact, Obama incorporates Biblical references that emphasize the importance of “that single moment on the bus” (lines 57-58). In lines 33-35, Obama explains that Parks and the other protestors are “driven by a solemn determination to affirm their God-given dignity” and he also compares their victory to the fall the “ancient walls of Jericho” (line 43). By of including these Biblical references, Obama suggests that Parks’ action on the bus did more than correct personal or political wrongs; it also corrected moral and spiritual wrongs. Although Parks had no political power or fortune, she was able to restore a moral balance in our world.

Toward the end of the speech, Obama states that change happens “not mainly through the exploits of the famous and the powerful, but through the countless acts of often anonymous courage and kindness” (lines 78-81). Through carefully chosen diction that portrays her as a quiet, regular person and through lists and Biblical references that highlight the huge impacts of her action, Obama illustrates exactly this point. He wants us to see that, just like Parks, the small and meek can change the world for the better.

AP Lang Rhetorical Analysis Essay Example: Why It Works

We would give the AP Lang Rhetorical Analysis essay above a score of 6 out of 6 because it fully satisfies the essay’s 3 rubric categories: Thesis, Evidence and Commentary, and Sophistication . Let’s break down what this student did:

The thesis of this essay appears in the last line of the first paragraph:

“ Through the use of diction that portrays Parks as quiet and demure, long lists that emphasize the extent of her impacts, and Biblical references, Obama suggests that all of us are capable of achieving greater good, just as Parks did .”

This student’s thesis works because they make a clear argument about Obama’s rhetorical choices. They 1) list the rhetorical choices that will be analyzed in the rest of the essay (the italicized text above) and 2) include an argument someone else might disagree with (the bolded text above).

Evidence and Commentary:

This student includes substantial evidence and commentary. Things they do right, per the AP Lang Rhetorical Analysis rubric:

  • They include lots of specific evidence from the text in the form of quotes.
  • They incorporate 3 different types of evidence (diction, long lists, Biblical references).
  • After including evidence, they offer an interpretation of what the evidence means and explain how the evidence contributes to their overarching argument (aka their thesis).

Sophistication

This essay achieves sophistication according to the AP Lang Rhetorical Analysis essay rubric in a few key ways:

  • This student provides an introduction that flows naturally into the topic their essay will discuss. Before they get to their thesis, they tell us that Obama portrays Parks as a “regular person” setting up their main argument: Obama wants all regular people to aspire to do good in the world just as Rosa Parks did.
  • They organize evidence and commentary in a clear and cohesive way. Each body paragraph focuses on just one type of evidence.
  • They explain how their evidence is significant. In the final sentence of each body paragraph, they draw a connection back to the overarching argument presented in the thesis.
  • All their evidence supports the argument presented in their thesis. There is no extraneous evidence or misleading detail.
  • They consider nuances in the text. Rather than taking the text at face value, they consider what Obama’s rhetorical choices imply and offer their own unique interpretation of those implications.
  • In their final paragraph, they come full circle, reiterate their thesis, and explain what Obama’s rhetorical choices communicate to readers.
  • Their sentences are clear and easy to read. There are no grammar errors or misused words.

AP Lang Rhetorical Analysis Essay—More Resources

Looking for more tips to help your master your AP Lang Rhetorical Analysis Essay? Brush up on 20 Rhetorical Devices High School Students Should Know and read our Tips for Improving Reading Comprehension . If you’re ready to start studying for another part of the AP English Exam, find more expert tips in our How to Write the AP Lang Synthesis blog post.

Considering what other AP classes to take? Read up on the Hardest AP Classes .

  • High School Success

' src=

Christina Wood

Christina Wood holds a BA in Literature & Writing from UC San Diego, an MFA in Creative Writing from Washington University in St. Louis, and is currently a Doctoral Candidate in English at the University of Georgia, where she teaches creative writing and first-year composition courses. Christina has published fiction and nonfiction in numerous publications, including The Paris Review , McSweeney’s , Granta , Virginia Quarterly Review , The Sewanee Review , Mississippi Review , and Puerto del Sol , among others. Her story “The Astronaut” won the 2018 Shirley Jackson Award for short fiction and received a “Distinguished Stories” mention in the 2019 Best American Short Stories anthology.

  • 2-Year Colleges
  • Application Strategies
  • Best Colleges by Major
  • Best Colleges by State
  • Big Picture
  • Career & Personality Assessment
  • College Essay
  • College Search/Knowledge
  • College Success
  • Costs & Financial Aid
  • Dental School Admissions
  • Extracurricular Activities
  • Graduate School Admissions
  • High Schools
  • Law School Admissions
  • Medical School Admissions
  • Navigating the Admissions Process
  • Online Learning
  • Private High School Spotlight
  • Summer Program Spotlight
  • Summer Programs
  • Test Prep Provider Spotlight

College Transitions Sidebar Block Image

“Innovative and invaluable
use this book as your college lifeline.”

— Lynn O'Shaughnessy

Nationally Recognized College Expert

College Planning in Your Inbox

Join our information-packed monthly newsletter.

I am a... Student Student Parent Counselor Educator Other First Name Last Name Email Address Zip Code Area of Interest Business Computer Science Engineering Fine/Performing Arts Humanities Mathematics STEM Pre-Med Psychology Social Studies/Sciences Submit

The Writing Center of Princeton

Thesis Statement Formula for AP English Rhetorical Analysis Essays

A good thesis statement presents your topic to the reader and indicates how you will interpret the significance of the subject matter discussed in your essay. Think of it as a kind of road map, designed to help the reader know what to expect in the essay.

But an AP rhetorical analysis thesis statement is like nothing you’ve ever had to write in school before. Unlike other kinds of thesis statements, a rhetorical analysis thesis statement demands that you do three things:

Identify the rhetorical devices you will analyze in your essay

Identify the impact of those devices of the effectiveness of the text

Identify the author, genre, and name of the text

Sound daunting? Not to worry!

The below, fill-in-the-blank thesis statement formula, designed for use when writing rhetorical analysis essays, will make your life simpler, easier, and more successful!

how to write an ap lang rhetorical analysis thesis

Have a language expert improve your writing

Run a free plagiarism check in 10 minutes, generate accurate citations for free.

  • Knowledge Base
  • How to write a rhetorical analysis | Key concepts & examples

How to Write a Rhetorical Analysis | Key Concepts & Examples

Published on August 28, 2020 by Jack Caulfield . Revised on July 23, 2023.

A rhetorical analysis is a type of essay  that looks at a text in terms of rhetoric. This means it is less concerned with what the author is saying than with how they say it: their goals, techniques, and appeals to the audience.

Instantly correct all language mistakes in your text

Upload your document to correct all your mistakes in minutes

upload-your-document-ai-proofreader

Table of contents

Key concepts in rhetoric, analyzing the text, introducing your rhetorical analysis, the body: doing the analysis, concluding a rhetorical analysis, other interesting articles, frequently asked questions about rhetorical analysis.

Rhetoric, the art of effective speaking and writing, is a subject that trains you to look at texts, arguments and speeches in terms of how they are designed to persuade the audience. This section introduces a few of the key concepts of this field.

Appeals: Logos, ethos, pathos

Appeals are how the author convinces their audience. Three central appeals are discussed in rhetoric, established by the philosopher Aristotle and sometimes called the rhetorical triangle: logos, ethos, and pathos.

Logos , or the logical appeal, refers to the use of reasoned argument to persuade. This is the dominant approach in academic writing , where arguments are built up using reasoning and evidence.

Ethos , or the ethical appeal, involves the author presenting themselves as an authority on their subject. For example, someone making a moral argument might highlight their own morally admirable behavior; someone speaking about a technical subject might present themselves as an expert by mentioning their qualifications.

Pathos , or the pathetic appeal, evokes the audience’s emotions. This might involve speaking in a passionate way, employing vivid imagery, or trying to provoke anger, sympathy, or any other emotional response in the audience.

These three appeals are all treated as integral parts of rhetoric, and a given author may combine all three of them to convince their audience.

Text and context

In rhetoric, a text is not necessarily a piece of writing (though it may be this). A text is whatever piece of communication you are analyzing. This could be, for example, a speech, an advertisement, or a satirical image.

In these cases, your analysis would focus on more than just language—you might look at visual or sonic elements of the text too.

The context is everything surrounding the text: Who is the author (or speaker, designer, etc.)? Who is their (intended or actual) audience? When and where was the text produced, and for what purpose?

Looking at the context can help to inform your rhetorical analysis. For example, Martin Luther King, Jr.’s “I Have a Dream” speech has universal power, but the context of the civil rights movement is an important part of understanding why.

Claims, supports, and warrants

A piece of rhetoric is always making some sort of argument, whether it’s a very clearly defined and logical one (e.g. in a philosophy essay) or one that the reader has to infer (e.g. in a satirical article). These arguments are built up with claims, supports, and warrants.

A claim is the fact or idea the author wants to convince the reader of. An argument might center on a single claim, or be built up out of many. Claims are usually explicitly stated, but they may also just be implied in some kinds of text.

The author uses supports to back up each claim they make. These might range from hard evidence to emotional appeals—anything that is used to convince the reader to accept a claim.

The warrant is the logic or assumption that connects a support with a claim. Outside of quite formal argumentation, the warrant is often unstated—the author assumes their audience will understand the connection without it. But that doesn’t mean you can’t still explore the implicit warrant in these cases.

For example, look at the following statement:

We can see a claim and a support here, but the warrant is implicit. Here, the warrant is the assumption that more likeable candidates would have inspired greater turnout. We might be more or less convinced by the argument depending on whether we think this is a fair assumption.

Here's why students love Scribbr's proofreading services

Discover proofreading & editing

Rhetorical analysis isn’t a matter of choosing concepts in advance and applying them to a text. Instead, it starts with looking at the text in detail and asking the appropriate questions about how it works:

  • What is the author’s purpose?
  • Do they focus closely on their key claims, or do they discuss various topics?
  • What tone do they take—angry or sympathetic? Personal or authoritative? Formal or informal?
  • Who seems to be the intended audience? Is this audience likely to be successfully reached and convinced?
  • What kinds of evidence are presented?

By asking these questions, you’ll discover the various rhetorical devices the text uses. Don’t feel that you have to cram in every rhetorical term you know—focus on those that are most important to the text.

The following sections show how to write the different parts of a rhetorical analysis.

Like all essays, a rhetorical analysis begins with an introduction . The introduction tells readers what text you’ll be discussing, provides relevant background information, and presents your thesis statement .

Hover over different parts of the example below to see how an introduction works.

Martin Luther King, Jr.’s “I Have a Dream” speech is widely regarded as one of the most important pieces of oratory in American history. Delivered in 1963 to thousands of civil rights activists outside the Lincoln Memorial in Washington, D.C., the speech has come to symbolize the spirit of the civil rights movement and even to function as a major part of the American national myth. This rhetorical analysis argues that King’s assumption of the prophetic voice, amplified by the historic size of his audience, creates a powerful sense of ethos that has retained its inspirational power over the years.

The body of your rhetorical analysis is where you’ll tackle the text directly. It’s often divided into three paragraphs, although it may be more in a longer essay.

Each paragraph should focus on a different element of the text, and they should all contribute to your overall argument for your thesis statement.

Hover over the example to explore how a typical body paragraph is constructed.

King’s speech is infused with prophetic language throughout. Even before the famous “dream” part of the speech, King’s language consistently strikes a prophetic tone. He refers to the Lincoln Memorial as a “hallowed spot” and speaks of rising “from the dark and desolate valley of segregation” to “make justice a reality for all of God’s children.” The assumption of this prophetic voice constitutes the text’s strongest ethical appeal; after linking himself with political figures like Lincoln and the Founding Fathers, King’s ethos adopts a distinctly religious tone, recalling Biblical prophets and preachers of change from across history. This adds significant force to his words; standing before an audience of hundreds of thousands, he states not just what the future should be, but what it will be: “The whirlwinds of revolt will continue to shake the foundations of our nation until the bright day of justice emerges.” This warning is almost apocalyptic in tone, though it concludes with the positive image of the “bright day of justice.” The power of King’s rhetoric thus stems not only from the pathos of his vision of a brighter future, but from the ethos of the prophetic voice he adopts in expressing this vision.

Receive feedback on language, structure, and formatting

Professional editors proofread and edit your paper by focusing on:

  • Academic style
  • Vague sentences
  • Style consistency

See an example

how to write an ap lang rhetorical analysis thesis

The conclusion of a rhetorical analysis wraps up the essay by restating the main argument and showing how it has been developed by your analysis. It may also try to link the text, and your analysis of it, with broader concerns.

Explore the example below to get a sense of the conclusion.

It is clear from this analysis that the effectiveness of King’s rhetoric stems less from the pathetic appeal of his utopian “dream” than it does from the ethos he carefully constructs to give force to his statements. By framing contemporary upheavals as part of a prophecy whose fulfillment will result in the better future he imagines, King ensures not only the effectiveness of his words in the moment but their continuing resonance today. Even if we have not yet achieved King’s dream, we cannot deny the role his words played in setting us on the path toward it.

If you want to know more about AI tools , college essays , or fallacies make sure to check out some of our other articles with explanations and examples or go directly to our tools!

  • Ad hominem fallacy
  • Post hoc fallacy
  • Appeal to authority fallacy
  • False cause fallacy
  • Sunk cost fallacy

College essays

  • Choosing Essay Topic
  • Write a College Essay
  • Write a Diversity Essay
  • College Essay Format & Structure
  • Comparing and Contrasting in an Essay

 (AI) Tools

  • Grammar Checker
  • Paraphrasing Tool
  • Text Summarizer
  • AI Detector
  • Plagiarism Checker
  • Citation Generator

The goal of a rhetorical analysis is to explain the effect a piece of writing or oratory has on its audience, how successful it is, and the devices and appeals it uses to achieve its goals.

Unlike a standard argumentative essay , it’s less about taking a position on the arguments presented, and more about exploring how they are constructed.

The term “text” in a rhetorical analysis essay refers to whatever object you’re analyzing. It’s frequently a piece of writing or a speech, but it doesn’t have to be. For example, you could also treat an advertisement or political cartoon as a text.

Logos appeals to the audience’s reason, building up logical arguments . Ethos appeals to the speaker’s status or authority, making the audience more likely to trust them. Pathos appeals to the emotions, trying to make the audience feel angry or sympathetic, for example.

Collectively, these three appeals are sometimes called the rhetorical triangle . They are central to rhetorical analysis , though a piece of rhetoric might not necessarily use all of them.

In rhetorical analysis , a claim is something the author wants the audience to believe. A support is the evidence or appeal they use to convince the reader to believe the claim. A warrant is the (often implicit) assumption that links the support with the claim.

Cite this Scribbr article

If you want to cite this source, you can copy and paste the citation or click the “Cite this Scribbr article” button to automatically add the citation to our free Citation Generator.

Caulfield, J. (2023, July 23). How to Write a Rhetorical Analysis | Key Concepts & Examples. Scribbr. Retrieved April 15, 2024, from https://www.scribbr.com/academic-essay/rhetorical-analysis/

Is this article helpful?

Jack Caulfield

Jack Caulfield

Other students also liked, how to write an argumentative essay | examples & tips, how to write a literary analysis essay | a step-by-step guide, comparing and contrasting in an essay | tips & examples, unlimited academic ai-proofreading.

✔ Document error-free in 5minutes ✔ Unlimited document corrections ✔ Specialized in correcting academic texts

logo-type-white

APÂź English Language

3 apÂź english language rhetorical essay strategies.

  • The Albert Team
  • Last Updated On: March 1, 2022

3_ap_english_language rhetorical essay strategies

The APÂź English Language rhetorical essay can be nightmare inducing for some APÂź students, but there is no need for fear. In this exam review we will lay out helpful strategies to get you through the rhetorical essays in no time.

Rhetorical Strategy #1: Dissecting the Prompt

The first rhetorical essay strategy is to dissect the prompt. Understanding what the rhetorical essay wants from you is essential. It is important for you to read the prompt carefully for every essay, but critical reading is even more essential to the rhetorical essay. Your rhetorical prompt that you will be given for the APÂź English Language exam will contain two elements. The first element is the concrete task that the prompt is asking of you, which is always to analyze the passage that follows. The second part of the prompt is a more abstract task that is not directly asked for in the prompt, but it is implied. By completely understanding both parts of the prompt, you will be able to give a complete essay that will get you to a higher score.

One example of a prompt from an APÂź English Language rhetorical essay is this one from the 2008 exam . The prompt reads:

“In the following passage from The Great Influenza, an account of the 1918 flu epidemic, author John M. Barry writes about scientists and their research. Read the passage carefully. Then, in a well-written essay, analyze how Barry uses rhetorical strategies.”

Here you can see the concrete task that the examiners are asking. They want you to analyze the passage for rhetorical strategies; however, you must figure out what you are analyzing the passage for. That is the more abstract concept that you need to dissect the prompt to find. In the case of Barry’s passage you will need to analyze how he uses rhetorical strategies in order to portray scientific research. We know this, because if you look at the prompt, it specifically states what Barry did in his work, which was to write about science and research. That is your abstract task.

Once you have found your concrete task and your abstract task, a great strategy is to write it down to keep you focused throughout your essay . Using the example above this would look like the following:

Analyze how Barry uses rhetorical strategies in order to portray scientific research.

That sentence is what you must follow when writing your essay, and if you successfully keep to this task, then you will move closer to that high score.

Rhetorical Essay Strategy #2: Stick to the Format

This next rhetorical essay strategy is the key to great organization and structure that will put your test anxiety to bed. There is a simple paragraph structure for the body paragraphs of the AP¼ English Language rhetorical essay that will allow you to think, write, and score higher, faster. You need to begin each body paragraph with an assertion or claim. That is the point that you are trying to make clear to your audience what you will be proving. A great example of this is from the 2006 AP¼ English Language rhetorical essay. Below is student 2B’s opening sentence for her first body paragraph.

“The diction of the passage fully relays Hazlitt’s position about money ( student 2B ).”

You can see how the student directly asserts what he or she will be proving in this statement. The next step in constructing your body paragraph is to give one to two pieces of textual evidence. Be sure to state why these quotations relate back to your claim, otherwise they will be deemed irrelevant by the examiners. An example of this is the next sentence in student 2B’s body paragraph about diction. Here, the student brings in elements from the text to support his or her claim about Hazlitt using diction.

“’Rejected’, ‘contempt’, ‘disparaged’, ‘scrutinized’, ‘irksome’, ‘deprived’, ‘assailed’, ‘chagrin’; the endless repetition of such discouragement shows just how emphatically Hazlitt money is requisite for happy life (student 2B).”

The final part of this strategy for conquering the body paragraphs of your rhetorical essays is to end those body paragraphs with a thorough analysis. This is the aspect of the exam where you can put your way of looking at the text into your essay.

An example of this is at the end of student 2B’s body paragraph where he or she states, “The irony of the last sentences is negative, conveying the utter hopelessness of one without money. Though one may have none in life, pitiless men will continue to mock one’s circumstances even after death! (student 2B)”

This analysis of the text adds to the textual examples above and continues to bring in new logic from the student.

When this format of a body paragraph is followed, then it is extremely effective. The essay becomes clear, assertive, and easy to follow for the examiners. Follow this rhetorical essay strategy and you are even closer to getting that 5 on the exam.

Rhetorical Essay Strategy #3: LORA

As you are looking at your APÂź English Language rhetorical essay prompt and passage it is important to remember the mnemonic device, LORA. LORA stands for Language, Organization, and Rhetorical Appeals. These elements will help you form your argument.

When you read through your passage you want to think about how the author is utilizing language. Is he or she using figurative language effectively? Is there imagery within the passage? Does the diction of the passage make it more rhetorically persuasive? You should not use all of these, but picking one and analyzing it clearly in one paragraph will keep you focused on how the author uses rhetoric, which is the main task of this essay.

An example of this was in the 2006 AP¼ English Language rhetorical essay. Student 2A begins his or her first body paragraph with, “One of Hazlitt’s most effective methods of promoting the importance of money is his strong diction (student 2A).” This student begins his or her essay with focusing on diction as how the language is used. He or she then goes on to explain why diction betters Hazlitt’s argument, which is exactly what you must do for your own rhetorical essay.

The organization of the author is the next part of your answer to the prompt. You want to look at how the author organized his or her ideas within the passage to support his or her own argument. By pointing out the organization, or structure, of the work and how it adds to the overall persuasiveness, you will bring two of the three most important elements of rhetoric together in your essay.

After organization you need to look at the rhetoric appeals. You may know them by the names logos, pathos, and ethos. It is suggested that you cover as many of these as possible; however, if time does not permit or if the passage uses one more than the other, then you should focus on one appeal.

One example of using pathos in an essay is from student 2A from the 2006 prompt. “Hazlitt plays on the audience’s heartstrings for more than enough time to convince them of the importance of having money (student 2A).” While it would have been better for the student to directly say that this is pathos, he or she does thoroughly explain the appeal to the passions, or pathos.

Key Takeaways

When taking the APÂź English Language rhetoric essay you just need to remember these three rhetorical essay strategies: dissect the prompt, follow the format, and always include LORA. If you can follow them, then you are already on your way to a 5 on the APÂź English Language exam .

Let’s put everything into practice. Try this APÂź English Language practice question:

Rhetorical Considerations APÂź English Language Practice Question

Looking for more APÂź English Language practice?

Check out our other articles on APÂź English Language .

You can also find thousands of practice questions on Albert.io. Albert.io lets you customize your learning experience to target practice where you need the most help. We’ll give you challenging practice questions to help you achieve mastery of APÂź English Language.

Start practicing here .

Are you a teacher or administrator interested in boosting APÂź English Language student outcomes?

Learn more about our school licenses here .

Interested in a school license?​

Popular posts.

APÂź Physics I score calculator

APÂź Score Calculators

Simulate how different MCQ and FRQ scores translate into APÂź scores

how to write an ap lang rhetorical analysis thesis

APÂź Review Guides

The ultimate review guides for APÂź subjects to help you plan and structure your prep.

how to write an ap lang rhetorical analysis thesis

Core Subject Review Guides

Review the most important topics in Physics and Algebra 1 .

how to write an ap lang rhetorical analysis thesis

SATÂź Score Calculator

See how scores on each section impacts your overall SATÂź score

how to write an ap lang rhetorical analysis thesis

ACTÂź Score Calculator

See how scores on each section impacts your overall ACTÂź score

how to write an ap lang rhetorical analysis thesis

Grammar Review Hub

Comprehensive review of grammar skills

how to write an ap lang rhetorical analysis thesis

APÂź Posters

Download updated posters summarizing the main topics and structure for each APÂź exam.

Interested in a school license?

how to write an ap lang rhetorical analysis thesis

Bring Albert to your school and empower all teachers with the world's best question bank for: ➜ SAT¼ & ACT¼ ➜ AP¼ ➜ ELA, Math, Science, & Social Studies aligned to state standards ➜ State assessments Options for teachers, schools, and districts.

AP Âź Lang teachers: looking to help your students improve their rhetorical analysis essays?

Coach Hall Writes

clear, concise rhetorical analysis instruction.

How to Write a Rhetorical Analysis Thesis

November 20, 2021 by Beth Hall

One of the first steps of writing a rhetorical analysis essay is knowing how to write a rhetorical analysis thesis.

Rhetorical analysis thesis statements can seem intimidating, but they do not have to be.

While the thesis is a small portion of an essay, it carries significant weight and impact, especially on the APÂź Lang exam. For example, on APÂź Lang rubric, a defensible thesis is one out of six possible points.

So, what is a defensible thesis and how do you write one for a rhetorical analysis essay?

A defensible thesis means that the thesis or position can be justified, proven, or defended.

You can craft a rhetorical analysis thesis statement with the following steps:

Step 1: As you are reading the passage, look for strategies or choices the author utilizes. Ask: What rhetorical choices does the writer/speaker make? (ie. juxtaposition, allusion, etc) This will be the basis of your thesis statement.

Step 2: Mention the author’s purpose in the thesis. Ask: Why did he/she make these choices? Why did he/she write this?

Step 3: Consider the effect on the audience. This step is not mandatory or always appropriate, but it can strengthen the thesis. The effect is looking at the author’s call to action. Ask: How does he/she want the audience to think/act?

how-to-write-a-rhetorical-analysis-thesis

Now that you understand the basis of a thesis statement, let’s talk about where this thesis goes in the essay.

The thesis is best placed in the introductory paragraph. By placing it in the introduction, it gives you a direction for your writing (and often where readers go looking for the thesis). The introduction contains the hook, context, and thesis statement. Often, the context and the thesis are combined together (look at the example below). The context identifies the specific passage you are talking about in your essay.

You can write only a thesis statement for an introductory paragraph if you are short on time, but it is better to have a well-developed introduction. If you want to know more about writing an introduction, you can watch the video here.

Let’s put this information together and look at an example of a thesis statement.

In Leonid Fridman’s passionate article “America Needs its Nerds,” ← context

he defines “geek” and contrasts America with other industrialized nations to develop his argument that America values athletes more than intellectuals. ← thesis

By doing so, Fridman urges readers to reprioritize the current social hierarchy. ← Effect

If you are feeling unsure about thesis statements or need a place to start, sentence frames are a great way to begin a thesis statement. Below are several sentence frames and examples to help you navigate thesis statements.

In SPEAKER/WRITER’S (tone) speech/letter/article (to AUDIENCE), he/she uses ___ and ____ to PURPOSE.

Note: The blanks in this sentence frame should be choices or strategies (nouns). For example, “he uses repetition and juxtaposition to
” Saying “uses” and then a device is rather simple. However, this sentence frame can lead to a defensible thesis. Once you understand this style of thesis writing, you can try more advanced styles.

In SPEAKER/WRITER’S (tone) speech/letter/article (to AUDIENCE), he/she ____ and ____ to PURPOSE.

Example: In his patriotic speech to Congress, President Roosevelt repeats “attacked” and “deliberate” as well as appeals to patriotism in order to convince Congress to declare war on Japan.

Example: In his patriotic speech to Congress, President Roosevelt repeatedly emphasizes the deliberate nature of the attack on Pearl Harbor and appeals to patriotism in order to convince Congress to declare war on Japan.

When you are ready to begin writing thesis statements on your own, remember to keep the following items in mind:

  • A thesis identifies the strategies / choices AND purpose. Without both of these, it is not a defensible thesis.
  • A thesis does not restate the prompt. Use the prompt as a guide, not as a thesis.
  • A thesis answers the prompt. This may seem obvious, but it can be easy to get caught up in writing and lose track of your goal

Looking for more tips about how to write a rhetorical analysis essay, check out this post here.

APÂź Lang Teachers

Looking to help your students improve their rhetorical analysis essays?

Latest on Instagram

how to write an ap lang rhetorical analysis thesis

Shop My TPT Store

Get in touch with us

Are you sure you want to logout?

Study abroad.

bannerAd

AP Lang Rhetorical Analysis Essay

AP language exams are held for numerous subjects. Students have to choose a specific subject and get higher scores. The higher the score, the higher the chance of pursuing the best 700 colleges or universities overall in the world, including the United States and Canada.

When it comes to AP English language, it involves a section called rhetorical analysis essay. This is a part of three free-response essays that have to be answered within 2 hours and 15 minutes from the overall 3 hours 15 minutes exam. 

parallel

If you are taking the AP Lang exam this year, guidance on how to answer this part will be useful. This article is specially curated to help you score the best. Read on to learn more about the AP language rhetorical analysis essay and get an idea of how to prepare for the associated exam successfully.

What is the AP Lang Rhetorical Analysis Essay ?

AP elaborated that the Advanced Placement is the exam conducted by the College Board in the United States of America. The exam is generally offered at the high school level and helps students to pursue higher education at the university level. The exam holds two sections: MCQs and Rhetorical Essay. MCQ sections are for 1 hour, and the rest of the time is for the essay section. The free-response essay holds three essays: rhetorical analysis essay, synthesis essay, and argumentative essay. 

parallel

  • The rhetorical analysis essay in the AP Lang exam involves students having to discuss how the authors’ contribution to the passage gives a theme or meaning. 
  • A synthesis essay involves students creating arguments on the passage or piece of information delivered to them.
  • An argumentative essay requires students to pick a side ‘for or against ‘ for an argument or debate.

Since we are here to discuss rhetorical analysis essays in AP language, you must know the essay is added to test students’ ability to analyze and interpret the deeper meaning in the provided passage. Through rhetorical essays, the examiner examines how students connect with the author’s style of writing and syntax within 40 minutes. Some students may face challenges while dealing with this essay because it requires a better understanding of rhetorical strategies and the method to apply them.

Tips to Write a Rhetorical Analysis Essay AP Lang ?

Since a rhetorical essay is quite tricky in comparison to other essays, it requires certain tips for a better approach to answering. Let us explore the method of writing a rhetorical essay for the AP Lang exam that helps you score well.

parallel

  • Outline Essay Prior to Writing

Not only good content but providing a readable structure is an important part of the rhetorical analysis essay AP Lang . First, you must read the passage thoroughly and develop a brief outline or key points before writing the essay. This helps you write with respect to the chronology of the given passage and maintain the flow of writing.

  • Understand Rhetorical Strategies

You aren’t alone in thinking about where to start writing a rhetorical essay. Many students face the same and are required to learn and implement rhetorical strategies. First, understand what rhetoric actually means. It refers to language sensibly chosen and structured for an impressive effect on the audience. This involves persuasive appeal, logical fallacies, and syntax such as anaphora, anthesis, parallelism, and so on. There is a vast range of elements that you can assess in the provided essay and develop a strong grasp with consistent practice.

parallel

  • Make your Essay Well-Structured

Sometimes, students understand the rhetorical strategies but still get confused about where to initiate. It is recommended to start with an introduction that delivers the purpose of your writing. In the last introductory line, you must talk about the rhetorical strategies you will discuss in the piece. However, there are so many styles, syntax, and tones. You must be specific while listing them and then move to develop a body paragraph. 

Now, you have to collect all the rhetorical strategies you mentioned in the introduction to discuss your point of view chronologically. Be specific while discussing strategy, as only the crucial ones among them must be discussed. Never forget to cite the line from the original passage. Your write-up must maintain the flow and should include relativity among the paragraphs. End the rhetorical essay by summarising key points.

parallel

  • Never Forget to Explain your Examples

Do not just state the examples or deliver statements like ‘this is an example of pathos or logos.’ It is advisable to explain the example you have listed in context to the rhetorical elements you have mentioned and how it aids the author in their viewpoint. Stay detailed yet precise while writing the rhetorical analysis essay AP Lang .

AP Lang Rhetorical Analysis Essay Rubric

Did you know that the rhetorical analysis essay AP Lang is graded into three rubric categories? Students must pay heed to the categories, as the examiner seeks specific things in each of them. Also, learn about some dos and don’ts to score well.

parallel

When it comes to grading the thesis of the rhetorical analysis essay AP Lang , there is nothing nebulous. Either you get one point, or you lose one. Thesis points help you get close to higher scores, and thus, you must be mindful of the following points.

parallel

  • Developing an argument states that your interpretation leads to a risk of disagreement. Thus, your thesis statements must be in context with the author’s rhetorical choice. 
  • If you create a phrase in your mind initiating with ‘I think that..’, your phrase should not go in a negative argumentative direction.
  • Never provide a thesis with summaries but not an argument.
  • Try not to provide the thesis with repeated prompts.

Evidence is provided to prove the argumentative context in the thesis. This rubric category is graded from 0 to 4 points. To score higher, you must follow the points below.

  • You must aim to provide multiple types of evidence in your argumentative thesis.
  • Each statement you provide must be backed up with evidence in context to the text or arguments about the author’s theoretical choices.
  • Add more and more evidence, which must be specific.
  • Examine whether your evidence is linked with your overarching argument.
  • Deliver your interpretation and never rely on just quotes or phrases. 
  • Steer clear of generalization for text or author
  • Avoid quotes that speak for themselves. You must elaborate on the evidence you provided.
  • Sophistication

According to The College Board, this category holds 0 to 1 points. The higher grading in rhetorical analysis essays indicates the delivery of the sophistication of thoughts or a complex understanding of rhetorical elements. In this, you have to mainly focus on the number of semicolons you use and not the fancy terms. Here is what you need to follow:

  • Focus on delivering the right connection between the thesis and your evidence.
  • Create a brief framework and then proceed with writing the essay in the right flow to stay precise and clear in your piece.
  • Do not include arguments that you won’t be able to provide evidence for.
  • Ignore complex or fancy words or phrases that are hard to follow.

Ways to Improve Your Rhetorical Analysis Essay AP Lang

Now that you know what a rhetorical analysis essay is and how it is developed, you must be thinking about how tricky it is to understand the passage. Many find themselves in a completely blank position when it comes to where to begin during the exam. They find the rhetorical analysis essay AP Lang quite challenging to deal with and understand the author’s perspective and viewpoint. Practicing rhetorical strategies does not help if you do not have the right approach, which only comes from the field experts and their guidance.  Turito offers a platform where you get mentors to teach you all the concepts and approaches you apply in writing effective rhetorical essays in less time. Their course helps you get higher scores, which you can use to get college admission to your favorite college and country. Hey, you ambitious one, what are you waiting for? Contact us now and give your dreams a flight!

Frequently Asked Questions

What courses do turito offer.

Turito offers several courses to help you prepare. Here are some of them for your reference. 1. Foundation Course 2. IIT JEE 3. NEET 4. Study Abroad 5. Universal Program 6. PSAT 7. AP 8. IELTS

Can I ask queries on Turito?

Yes absolutely! Turito offers a platform where you can ask your concerns and queries. You will get answers from the field expert that can let your worries fade away.

Is Turito genuine?

Turito is students’ most trusted learning platform, among others. Turito believes in transparency and thus provides genuine feedback from students and parents on the official website. At Turito, we offer guaranteed success for hardworking students. Thus, there isn’t any negative feedback to hide. If you have any queries, you can write to us at [email protected]. Our experts will connect and assist you in no time.

AP Lang Rhetorical Analysis Essay

Relevant Articles

AP Lang Synthesis Essay

Steps to Draft AP Lang Synthesis Essay

The synthesis essay AP lang is part of the AP …

Steps to Draft AP Lang Synthesis Essay Read More »

AP Calculus AB Exam

Strategies for Success: A Complete Overview of the AP Calculus AB Exam

The AP Calculus AB Exam serves as an important element …

Strategies for Success: A Complete Overview of the AP Calculus AB Exam Read More »

Digital AP Exams

Preparing for Success: A Guide to the Features of Digital AP Exams

Digital  Advanced Placement (AP) exams bring about a huge change …

Preparing for Success: A Guide to the Features of Digital AP Exams Read More »

card img

With Turito Study Abroad

card img

Get an Expert Advice from Turito

card img

With Turito CAP.

card img

With Turito Coding.

card img

With Turito RoboNinja

card img

1-on-1 tutoring for the undivided attention

Calculate for all schools

Your chance of acceptance, your chancing factors, extracurriculars, how to master the ap english language and composition rhetorical analysis essay.

Hey guys, I'm currently prepping for the AP English Language and Composition exam, and I'm having trouble with the rhetorical analysis essay. What are some tips and resources I should check out to help me ace this part of the exam? Thanks!

Hi there! Rhetorical analysis essays can be challenging, but with focused practice and a solid approach, you can do well on this section. Here are some tips and resources to help you:

1. Understand the rhetorical triangle: When analyzing a text, remember the rhetorical triangle, which consists of the author, audience, and purpose. Familiarize yourself with various rhetorical strategies authors use to engage with their audience and achieve their purpose.

2. Read and annotate: Take time to read and annotate the given passage, underlining or highlighting important rhetorical devices, transitions, and anything else that stands out to you. This will help you form the basis of your analysis.

3. Create an outline: Before writing your essay, jot down the main points you want to cover and organize them in a logical fashion. This will help ensure that your essay is well-structured, flows smoothly, and effectively covers the prompt.

4. Incorporate textual evidence: Always quote or paraphrase specific textual evidence to support your analysis. Don't simply state that rhetorical devices were used; show how they were used by referring to specific instances in the passage.

5. Use precise, concise language: Clearly articulate your analysis in a direct, concise manner. Use specific language and active verbs when analyzing the author's effectiveness.

6. Practice with past prompts: To get better at rhetorical analysis essays, practice by reviewing past AP exam prompts and analyzing various texts. This will help you build up your analytical skillset and familiarize yourself with the types of questions you might encounter.

- College Board's AP English Language and Composition Course and Exam Description (CED): This official College Board document provides a detailed breakdown of the course content, as well as sample exam questions and essays. You can find this on their website.

- CollegeVine: Their blog has multiple articles related to AP English Language and Composition that share tips, strategies, and advice to score well on the exam.

- AP Classroom: If your school has access to the College Board's AP Classroom, there are numerous resources available for practice, including sample essays and prompts.

- Review books: Many reputable companies publish review books specifically tailored to the AP English Language and Composition exam. These books often contain practice questions, summaries of rhetorical strategies, and tips for essay-writing.

About CollegeVine’s Expert FAQ

CollegeVine’s Q&A seeks to offer informed perspectives on commonly asked admissions questions. Every answer is refined and validated by our team of admissions experts to ensure it resonates with trusted knowledge in the field.

AP English Language and Composition: Sample Rhetorical Analysis and Synthesis Questions

April 9, 2024.

AP English Language and Composition: Sample Rhetorical Analysis and Synthesis Questions

The Rhetorical Analysis and Synthesis Essays are two of the three essays you’ll need to write as part of the AP English Language and Composition Exam . Read on for a sample of each, as well as tips for how to answer them. 

AP English Language and Composition: Sample Rhetorical Analysis Question

Read the following passage published back in 1967 by The New York Times. Then write an essay in which you analyze how the structure of the passage and the use of language help convey the writer’s views.

Sample Question Instructions:

  • Respond to the prompt with a thesis that may establish a line of reasoning.
  • Select and use evidence to develop and support the line of reasoning.
  • Explain the relationship between the evidence and the thesis.
  • Demonstrate an understanding of the rhetorical situation.
  • Use appropriate grammar and punctuation in communicating the argument.

Americans and Western Europeans, in their sensitivity to lingering problems around them, tend to make science and progress their scapegoats. There is a belief that progress has precipitated widespread unhappiness, anxieties, and other social and emotional problems. Science is viewed as a cold mechanical discipline having nothing to do with human warmth and the human spirit. 

But to many of us from the nonscientific East, science does not have such repugnant associations. We are not afraid of it, nor are we disappointed by it. We know all too painfully that our social and emotional problems festered long before the age of technology. To us, science is warm and reassuring. It promises hope. It is helping us at long last gain some control over our persecutory environments, alleviating age-old problems—not only physical but also, and especially, problems of the spirit.

Shiraz, for example, a city in southern Iran, has long been renowned for its rose gardens and nightingales; its poets, Sadi and Hafiz; and its mystical, ascetic philosophy, Sufism. Much poetry has been written in glorification of the spiritual attributes of this oasis city. And to be sure, Shiraz is a green, picturesque town, with a quaint bazaar and refreshing gardens. But in this “romantic” city thousands of emotionally disturbed and mentally retarded men, women, and children were, until recently, kept in chains in stifling prison cells and lunatic asylums. 

Every now and again, some were dragged, screaming and pleading, to a courtyard and flogged for not behaving “normally.” But for the most part, they were made to sit against damp walls, their hands and feet locked in chains, and thus immobilized, without even a modicum of affection from their helpless families and friends, they sat for weeks and months and years—often all their lives. Pictures of these wretched men, women, and children can still be seen in this “city of poetry,” this “city with a spiritual way of life.” 

It was only recently that a wealthy young Shirazi who, against the admonitions of his family, had studied psychology at the University of Tehran and foreign universities, returned to Shiraz and after considerable struggle with city officials succeeded in opening a psychiatric clinic, the first in those regions. After still more struggle, he arranged to have the emotionally disturbed and the mentally retarded transferred from prison to their homes, to hospitals, and to his clinic, where he and his staff now attend them. 

They are fortunate. All over Asia and other backward areas, emotionally disturbed men and women are still incarcerated in these medieval dungeons called lunatic asylums. The cruel rejection and punishment are intended to teach them a lesson or help exorcise evil spirits. 

The West, still bogged down in its ridiculous romanticism, would like to believe that emotional disturbances, dope addiction, delinquency are all modern problems brought on by technological progress, and that backward societies are too spiritual and beautiful to need the ministrations of science. But while the West can perhaps afford to think this way, the people of backward lands cannot. . . . 

. . .The obstacles are awesome, the inertia too entrenched, the people’s suffering too anguished, their impatience too eruptive. Moreover, the total cultural reorganizations such as Asia and Africa are undergoing inevitably engender their own temporary dislocations and confusions. But their goals, the direction, remain constant. We are on the move, however awkwardly at first, to a saner, better world.

How to Answer the AP English Language and Composition Rhetorical Analysis Question

Go back to the original question, which asks you to analyze two features of the passage: (1) its structure, or organization, and (2) its language. The first aspect is fairly specific. As you read the passage, you need to observe what the author discusses first, second, third, and so on. Your essay should explain not only the order of ideas but the reasons the author may have chosen that order. 

The second part of the question is more general. It invites you to analyze the use of language, which may include the author’s choice of words (diction), syntax (word order), figures of speech, use of evidence (such as statistics or logical reasoning), sentence structure, rhythm, sound, tone, or just about any other characteristics of style and rhetoric you choose. 

Although the question directs you to write about two different aspects of the passage, the essay itself should be unified. That is, a good essay should not consist of, say, two disparate paragraphs, one exclusively devoted to structure and another to language. Rather, the essay should include material that shows the interrelationship of structure and language in the passage and how those elements contribute to the meaning and effect of the passage. This might be covered in a separate paragraph, or it could be woven into the overall fabric of the essay. 

Before you begin to write, read the passage at least twice: once for an overview and once as you write your analysis. You may notice early on that the opening paragraph contains generalizations about Westerners’ concepts of science and progress. Then the author contrasts the Western view of science and progress with the Eastern view. Immediately, you see that the author, by using the first-person pronoun (as in “many of us”) is speaking from the perspective of an Easterner. Consequently, his discussion of Eastern views is apt to come across as more well-informed, more authoritative, perhaps more personal. 

To support his position, the author gives an extended example—the city of Shiraz—to illustrate just how different the East is from the West. The description and vivid images of Shiraz memorably convey the idea that the “spiritual way of life” has a side to it that many Westerners don’t know about. This is the heart of the passage. The use of quotation marks around “romantic” and “city of poetry” is meant to point out the discrepancy between the idealized and real versions of Shiraz. 

Nearing the end, the author reiterates his initial contrast between West and East, with emphasis on the East. The last paragraph offers a generalized statement about conditions in Asia and Africa, reminding the reader of the contrast made at the very beginning of the passage. Tying the end to the beginning of the passage creates a sense of unity—a desirable feature in any piece of writing.

AP English Language and Composition: Sample Argument Question

The following paragraph is adapted from Mirror for Man, a book written by anthropologist Clyde Kluckhorn in the middle of the twentieth century. Read the passage carefully. Then, write an essay that examines the extent to which the author’s characterization of the United States holds true today. Use appropriate evidence to support your argument. 

Sample Question Instructions: 

  • Respond to the prompt with a thesis that may establish a line of reasoning. 
  • Select and use evidence to develop and support the line of reasoning. 
  • Explain the relationship between the evidence and the thesis. 
  • Demonstrate an understanding of the rhetorical situation. 

Technology is valued as the very basis of the capitalistic system. Possession of gadgets is esteemed as a mark of success to the extent that persons are judged not by the integrity of their characters or by the originality of their minds but by what they seem to be—so far as can be measured by their wealth or by the variety and material goods which they display. “Success” is measured by their investments, homes, and lifestyles— not by their number of mistresses as in some cultures.

How to Answer the AP English Language and Composition Argument Question

Whether you agree, disagree, or have mixed views on the content of the passage, your job is to write a convincing argument that expresses your opinion. Initially, the word argument may suggest conflict or confrontation. But rest assured that your essay need not be combative. Rather, make it a calmly-reasoned explanation of your opinion on a debatable subject. Your goal is to persuade the reader that your opinion, supported by examples, facts, and other appropriate evidence, is correct. 

If you have strong feelings about the topic, of course you should state them in your essay. But express them in calm, rational language. Be mindful that the essay should not be an emotional rant for or against the issue. 

Consider first whether you agree with Kluckhorn’s definition of “success.” Is it, as Kluckhorn asserts, measured by income and material possessions? Or do you think that a more accurate standard of success in today’s America should be determined by less tangible criteria—things such as happiness or self-respect? Or do you stand somewhere in between those two extremes? 

The actual position you take on the issue is less crucial than your ability to support it fully by drawing from your knowledge, background, experience, or observation. Regardless of your position, be sure to include more than one example. An argument that relies on a single example, however compelling, will fall flat. 

In the prompt, Kluckhorn’s notion of success seems to refer broadly to American society. Resist responding in kind. That is, a short essay shouldn’t focus on the whole of society but only on an identifiable segment—perhaps college-educated professionals or urban, blue- collar Americans. The point is that a narrowly focused essay on a limited topic will always turn out better than one that tries to cover too much ground in just a few paragraphs.

AP Biology Resources

  • About the AP Biology Exam
  • Top AP Biology Exam Strategies
  • Top 5 Study Topics and Tips for the AP Biology Exam
  • AP Biology Short Free-Response Questions
  • AP Biology Long Free-Response Questions

AP Psychology Resources

  • What’s Tested on the AP Psychology Exam?
  • Top 5 Study Tips for the AP Psychology Exam
  • AP Psychology Key Terms
  • Top AP Psychology Exam Multiple-Choice Question Tips
  • Top AP Psychology Exam Free Response Questions Tips
  • AP Psychology Sample Free Response Question

AP English Language and Composition Resources

  • What’s Tested on the AP English Language and Composition Exam?
  • Top 5 Tips for the AP English Language and Composition Exam
  • Top Reading Techniques for the AP English Language and Composition Exam
  • How to Answer the AP English Language and Composition Essay Questions 
  • AP English Language and Composition Exam Sample Essay Questions
  • AP English Language and Composition Exam Multiple-Choice Questions

AP Human Geography Resources

  • What’s Tested On the AP Human Geography Exam?
  • AP Human Geography FAQs
  • AP Human Geography Question Types and Strategies
  • Top 5 Study Tips for the AP Human Geography Exam

FOLLOW ALONG ON SOCIAL

pep

Find what you need to study

How to Answer AP Lang Rhetorical Analysis FRQs

3 min read ‱ december 14, 2021

Stephanie Kirk

Stephanie Kirk

Question 2: The Rhetorical Analysis FRQ

The AP Lang exam's free-response section is a writing section comprised of three essays within 2 hours and 15 minutes. The second essay is on Rhetorical Analysis, in which you look at a variety of nonfiction sources - newspaper articles, essays, letters, et cetera - and make connections between the author's rhetorical strategies and techniques, their central arguments, and their overall purpose.

Here's an overview on what you should expect and do in preparation for the AP exam!

General Information

Since the essays are 55% of your total score, it is helpful to have a plan before game day. Aim for around 40 minutes on reading this nonfiction text, creating a quick outline, and writing an essay that analyzes the author’s choices and their impact on meaning and purpose. While the exact prompt will vary, all prompt will follow this general description.

Source: For more information on the variety of FRQ 2 prompts, check out this list of past AP Lang free response questions.

https://firebasestorage.googleapis.com/v0/b/fiveable-92889.appspot.com/o/images%2F-QSrP0ePbGjIN.png?alt=media&token=f2154e91-d1dc-49c4-9598-6ffa2fe69e78

Break Down the Process

Before the test.

Here at Fiveable, we recognize the essays can be tricky.  For that reason, we suggest the following process to help you earn the FIVE you want on the AP Exam.

Know what to expect.   According to the 2019-2020 Course and Exam Description,

  • Learn a few key devices.   While naming devices isn’t required, it is helpful to know a few rhetorical terms so you can recognize them in the stimulus text and write about them clearly in your essay.  Learning the device also comes with knowing the effect it SHOULD have, and that means it will be easier for you in building your essay -- if you know WHAT is being done and HOW it should impact the audience, the WHY should be easy to find. We created this list of 40 rhetorical devices for AP Lang to help you out!
  • Practice Rhetorical Analysis.   You are going to be doing this in class, but have you considered the rhetorical situation of songs you love?  Practice RA outside of class in places where you see it -- songs, commercials, discussion with other people.  The rhetoric of songs can be fun because you can generally look up the author’s story of the song after you have made your own prediction.
  • Use the Rhetorical Precis.   The RP can be a great tool for your introduction paragraph because it offers a tool to organize the tone, purpose, and intended audience right from the start.  Next, it outlines the text, the rhetorical facilities employed, and the impact of the text as a whole.

During the Test

BAT the prompt to create your writing plan.  When writing the prompt, the College Board will give you plenty of information to jumpstart your analysis of the rhetorical situation.  Look for any background  you are given to help set the stage for the text, purpose, audience, or anything else.  Next, pay attention to the advice  you are given about what to do and how to do it.  Finally, process the task  at hand. Make sure you understand the question, including any implied sections you will need to answer to completely and correctly respond.

  • Read and annotate the text.
  • Clear flow with headings and subheadings.

After the Test

Don’t post anything about the content of the exam.

  • Post on social media and let us know how prepared you felt.
  • Celebrate your success!

FRQ #2 on Rhetorical Analysis might seem intimidating at first, but once you know your rhetorical devices, do some practice essays, and sleep tight the night before test day, you are going to be more than ready to take on the writing section of the AP Lang exam!

Fiveable

Stay Connected

© 2024 Fiveable Inc. All rights reserved.

APÂź and SATÂź are trademarks registered by the College Board, which is not affiliated with, and does not endorse this website.

What are your chances of acceptance?

Calculate for all schools, your chance of acceptance.

Duke University

Your chancing factors

Extracurriculars.

how to write an ap lang rhetorical analysis thesis

How to Write the AP Lang Synthesis Essay + Example

Do you know how to improve your profile for college applications.

See how your profile ranks among thousands of other students using CollegeVine. Calculate your chances at your dream schools and learn what areas you need to improve right now — it only takes 3 minutes and it's 100% free.

Show me what areas I need to improve

What’s Covered:

What is the ap lang synthesis essay, how will ap scores affect my college chances.

AP English Language and Composition, commonly known as AP Lang, is one of the most engaging and popular AP classes offered at most high schools, with over 535,000 students taking the class . AP Lang tests your ability to analyze written pieces, synthesize information, write rhetorical essays, and create cohesive and concrete arguments. However, the class is rather challenging as only 62% of students were able to score a three or higher on the exam. 

The AP Lang exam has two sections. The first consists of 45 multiple choice questions which need to be completed in an hour. This portion counts for around 45% of your total score. These questions ask students to analyze written pieces and answer questions related to each respective passage.  All possible answer choices can be found within the text, and no prior knowledge of literature is needed to understand the passages.

The second section contains three free-response questions to be finished in under two hours and 15 minutes. This section counts for 55% of your score and includes the synthesis essay, the rhetorical essay, and the argumentative essay.

  • The synthesis essay requires you to read 6-7 sources and create an argument using at least three sources.
  • The rhetorical analysis essay requires you to describe how a piece of writing evokes specific meanings and symbolism.
  • The argumentative essay requires you to pick a perspective of a debate and create an argument based on the evidence provided.

In this post, we will take a look at the AP Lang synthesis essay and discuss tips and tricks to master this part of the exam. We will also provide an example of a well-written essay for review.  

The AP Lang synthesis essay is the first of three essays included in the Free Response section of the AP Lang exam. The exam presents 6-7 sources that are organized around a specific topic, with two of those sources purely visual, including a single quantitative source (like a graph or pie chart). The remaining 4-5 sources are text-based, containing around 500 words each. It’s recommended that students spend an hour on this essay—15 minute reading period, 40 minutes writing, and 5 minutes of spare time to check over work.

Each synthesis essay has a topic that all the sources will relate to. A prompt will explaining the topic and provide some background, although the topics are usually broad so you will probably know something related to the issue. It will also present a claim that students will respond to in an essay format using information from at least three of the provided sources. You will need to take a stance, either agreeing or disagreeing with the position provided in the claim. 

According to the CollegeBoard, they are looking for essays that “combine different perspectives from sources to form a support of a coherent position.” This means that you must state your claim on the topic and highlight relationships between several sources that support your specific position on the topic. Additionally, you’ll need to cite clear evidence from your sources to prove your point.

The synthesis essay counts for six points on the AP Lang exam. Students can receive 0-1 points for writing a thesis statement, 0-4 based on the incorporation of evidence and commentary, and 0-1 points based on the sophistication of thought and demonstration of complex understanding.

While this essay seems extremely overwhelming, considering there are a total of three free-response essays to complete, with proper time management and practiced skills, this essay is manageable and straightforward. In order to enhance the time management aspect of the test to the best of your ability, it is essential to divide the essay up into five key steps.

Step 1: Analyze the Prompt

As soon as the clock starts, carefully read and analyze what the prompt asks from you. It might be helpful to markup the text to identify the most critical details. You should only spend around 2 minutes reading the prompt so you have enough time to read all the sources and figure out your argument. Don’t feel like you need to immediately pick your stance on the claim right after reading the prompt. You should read the sources before you commit to your argument.

Step 2: Read the Sources Carefully

Although you are only required to use 3 of the 6-7 sources provides, make sure you read ALL of the sources. This will allow you to better understand the topic and make the most educated decision of which sources to use in your essay. Since there are a lot of sources to get through, you will need to read quickly and carefully.

Annotating will be your best friend during the reading period. Highlight and mark important concepts or lines from each passage that would be helpful in your essay. Your argument will probably begin forming in your head as you go through the passages, so you will save yourself a lot of time later on if you take a few seconds to write down notes in the margins. After you’ve finished reading a source, reflect on whether the source defends, challenges, or qualifies your argument.

You will have around 13 minutes to read through all the sources, but it’s very possible you will finish earlier if you are a fast reader. Take the leftover time to start developing your thesis and organizing your thoughts into an outline so you have more time to write. 

Step 3: Write a Strong Thesis Statement 

In order to write a good thesis statement, all you have to do is decide your stance on the claim provided in the prompt and give an overview of your evidence. You essentially have three choices on how to frame your thesis statement: You can defend, challenge or qualify a claim that’s been provided by the prompt. 

  • If you are defending the claim, your job will be to prove that the claim is correct .
  • If you are challenging the claim, your job will be to prove that the claim is incorrect .
  • If you choose to qualify the claim, your job will be to agree to a part of the claim and disagree with another part of the claim. 

A strong thesis statement will clearly state your stance without summarizing the issue or regurgitating the claim. The CollegeBoard is looking for a thesis statement that “states a defensible position and establishes a line of reasoning on the issue provided in the prompt.”

Step 4: Create a Minimal Essay Outline

Developing an outline might seem like a waste of time when you are up against the clock, but believe us, taking 5-10 minutes to outline your essay will be much more useful in the long run than jumping right into the essay.

Your outline should include your thesis statement and three main pieces of evidence that will constitute each body paragraph. Under each piece of evidence should be 2-3 details from the sources that you will use to back up your claim and some commentary on how that evidence proves your thesis.

Step 5: Write your Essay

Use the remaining 30-35 minutes to write your essay. This should be relatively easy if you took the time to mark up the sources and have a detailed outline.  Remember to add special consideration and emphasis to the commentary sections of the supporting arguments outlined in your thesis. These sentences are critical to the overall flow of the essay and where you will be explaining how the evidence supports or undermines the claim in the prompt.

Also, when referencing your sources, write the in-text citations as follows: “Source 1,” “Source 2,” “Source 3,” etc. Make sure to pay attention to which source is which in order to not incorrectly cite your sources. In-text citations will impact your score on the essay and are an integral part of the process.

After you finish writing, read through your essay for any grammatical errors or mistakes before you move onto the next essay.

Here are six must-have tips and tricks to get a good score on the synthesis essay:

  • Cite at least four sources , even though the minimum requirement is three. Remember not to plagiarize and cite everything you use in your arguments.
  • Make sure to develop a solid and clear thesis . Develop a stable stance for the claim and stick with it throughout the entire paper.
  • Don’t summarize the sources. The summary of the sources does not count as an argument. 
  • You don’t necessarily have to agree with the sources in order to cite them. Using a source to support a counterargument is still a good use of a source.
  • Cite the sources that you understand entirely . If you don’t, it could come back to bite you in the end. 
  • Use small quotes , do not quote entire paragraphs. Make sure the quote does not disrupt the flow or grammar of the sentence you write. 

how to write an ap lang rhetorical analysis thesis

Discover your chances at hundreds of schools

Our free chancing engine takes into account your history, background, test scores, and extracurricular activities to show you your real chances of admission—and how to improve them.

Here is an example prompt and essay from 2019 that received 5 of the 6 total points available:

In response to our society’s increasing demand for energy, large-scale wind power has drawn attention from governments and consumers as a potential alternative to traditional materials that fuel our power grids, such as coal, oil, natural gas, water, or even newer sources such as nuclear or solar power. Yet the establishment of large-scale, commercial-grade wind farms is often the subject of controversy for a variety of reasons.

Carefully read the six sources, found on the AP English Language and Composition 2019 Exam (Question 1), including the introductory information for each source. Write an essay that synthesizes material from at least three of the sources and develops your position on the most important factors that an individual or agency should consider when deciding whether to establish a wind farm.

Source A (photo)

Source B (Layton)

Source C (Seltenrich)

Source D (Brown)

Source E (Rule)

Source F (Molla)

In your response you should do the following:

  • Respond to the prompt with a thesis presents a defensible position.
  • Select and use evidence from at least 3 of the provided sources to support your line of reasoning. Indicate clearly the sources used through direct quotation, paraphrase, or summary. Sources may be cited as Source A, Source B, etc., or by using the description in parentheses.
  • Explain how the evidence supports your line of reasoning.
  • Use appropriate grammar and punctuation in communicating your argument.

[1] The situation has been known for years, and still very little is being done: alternative power is the only way to reliably power the changing world. The draw of power coming from industry and private life is overwhelming current sources of non-renewable power, and with dwindling supplies of fossil fuels, it is merely a matter of time before coal and gas fuel plants are no longer in operation. So one viable alternative is wind power. But as with all things, there are pros and cons. The main factors for power companies to consider when building wind farms are environmental boon, aesthetic, and economic factors.

[2] The environmental benefits of using wind power are well-known and proven. Wind power is, as qualified by Source B, undeniably clean and renewable. From their production requiring very little in the way of dangerous materials to their lack of fuel, besides that which occurs naturally, wind power is by far one of the least environmentally impactful sources of power available. In addition, wind power by way of gearbox and advanced blade materials, has the highest percentage of energy retention. According to Source F, wind power retains 1,164% of the energy put into the system – meaning that it increases the energy converted from fuel (wind) to electricity 10 times! No other method of electricity production is even half that efficient. The efficiency and clean nature of wind power are important to consider, especially because they contribute back to power companies economically.

[3] Economically, wind power is both a boon and a bone to electric companies and other users. For consumers, wind power is very cheap, leading to lower bills than from any other source. Consumers also get an indirect reimbursement by way of taxes (Source D). In one Texan town, McCamey, tax revenue increased 30% from a wind farm being erected in the town. This helps to finance improvements to the town. But, there is no doubt that wind power is also hurting the power companies. Although, as renewable power goes, wind is incredibly cheap, it is still significantly more expensive than fossil fuels. So, while it is helping to cut down on emissions, it costs electric companies more than traditional fossil fuel plants. While the general economic trend is positive, there are some setbacks which must be overcome before wind power can take over as truly more effective than fossil fuels.

[4] Aesthetics may be the greatest setback for power companies. Although there may be significant economic and environmental benefit to wind power, people will always fight to preserve pure, unspoiled land. Unfortunately, not much can be done to improve the visual aesthetics of the turbines. White paint is the most common choice because it “[is] associated with cleanliness.” (Source E). But, this can make it stand out like a sore thumb, and make the gargantuan machines seem more out of place. The site can also not be altered because it affects generating capacity. Sound is almost worse of a concern because it interrupts personal productivity by interrupting people’s sleep patterns. One thing for power companies to consider is working with turbine manufacturing to make the machines less aesthetically impactful, so as to garner greater public support.

[5] As with most things, wind power has no easy answer. It is the responsibility of the companies building them to weigh the benefits and the consequences. But, by balancing economics, efficiency, and aesthetics, power companies can create a solution which balances human impact with environmental preservation.

More examples can be found here at College Board.

While AP Scores help to boost your weighted GPA, or give you the option to get college credit, AP Scores don’t have a strong effect on your admissions chances . However, colleges can still see your self-reported scores, so you might not want to automatically send scores to colleges if they are lower than a 3. That being said, admissions officers care far more about your grade in an AP class than your score on the exam.

Related CollegeVine Blog Posts

how to write an ap lang rhetorical analysis thesis

Developing a Thesis for Rhetorical Analysis: Strategies and Examples

This essay about the development of a thesis for rhetorical analysis provides a comprehensive exploration of strategies and examples drawn from various rhetorical discourses. It emphasizes the importance of identifying rhetorical devices, understanding contextual nuances, and maintaining clarity and specificity in thesis formulation. Through examples such as Martin Luther King Jr.’s “I Have a Dream” speech and contemporary political rhetoric, the essay illustrates how effective theses encapsulate the essence of discourse and its persuasive intent. It underscores the significance of honing analytical skills to navigate the complexities of rhetoric and shape discourse with precision.

How it works

As a diligent student of rhetoric, I find myself perpetually immersed in the intricate art of persuasion. Central to this endeavor is the crafting of a compelling thesis for rhetorical analysis. In this essay, I aim to dissect the strategies and exemplify the process of developing such a thesis, drawing from the rich tapestry of rhetorical discourse.

At its core, a rhetorical analysis thesis serves as the fulcrum upon which the entire analysis pivots. It encapsulates the essence of the discourse, delineating the rhetorical strategies employed by the author to convey their message effectively.

Crafting such a thesis requires a meticulous approach, intertwining keen observation with insightful interpretation.

One strategy essential to the formulation of a robust thesis is the identification of the rhetorical devices employed within the text. These devices serve as the building blocks of persuasion, enabling the author to wield language with precision and efficacy. From ethos, pathos, and logos to metaphor, simile, and irony, the rhetorical arsenal is vast and multifaceted. As a discerning student, it is imperative to unravel these devices, discerning their purpose and impact on the audience.

For instance, consider a thesis centered on Martin Luther King Jr.’s iconic “I Have a Dream” speech. By dissecting the text, one may identify King’s adept use of pathos through emotive language and vivid imagery. A thesis could thus assert: “Through the strategic deployment of pathos, Martin Luther King Jr. invokes a profound emotional resonance, galvanizing his audience towards the pursuit of racial equality.” Here, the thesis not only identifies the rhetorical strategy employed but also hints at its broader implications within the socio-political context of the Civil Rights Movement.

Furthermore, a nuanced understanding of rhetorical context is indispensable in thesis development. Context encompasses the myriad factors surrounding the discourse, including the historical backdrop, the intended audience, and the author’s overarching purpose. By contextualizing the text within its temporal and socio-cultural milieu, one can glean deeper insights into the rhetorical strategies at play.

Returning to our example of King’s speech, a contextualized thesis might elucidate: “Against the backdrop of pervasive racial injustice in 1960s America, Martin Luther King Jr. strategically harnesses the power of rhetoric to catalyze a movement for social change.” Here, the thesis not only acknowledges the historical context but also underscores the transformative potential of rhetorical discourse in precipitating societal shifts.

Moreover, a successful thesis for rhetorical analysis transcends mere identification of rhetorical elements; it delves into their cumulative effect on the audience and the broader discourse. This necessitates a nuanced analysis of tone, argumentative structure, and the author’s stance vis-à-vis the subject matter.

Consider the formulation of a thesis pertaining to a contemporary political speech. By dissecting the speaker’s tone, one may discern underlying nuances indicative of their rhetorical intent. A thesis could thus posit: “Through a combination of assertive rhetoric and appeals to national identity, the speaker constructs a compelling argument aimed at garnering public support for their policy agenda.” Here, the thesis not only scrutinizes the rhetorical strategies employed but also interprets their persuasive implications within the realm of public discourse.

In addition to strategic formulation, the efficacy of a rhetorical analysis thesis hinges on its clarity and specificity. A well-crafted thesis not only outlines the overarching rhetorical strategies but also articulates a clear analytical stance. Ambiguity or vagueness can dilute the potency of the thesis, rendering the subsequent analysis unfocused and disjointed.

To illustrate, consider a thesis that lacks specificity: “The author employs rhetorical devices to convey their message.” While technically accurate, this thesis falls short in providing a discernible analytical standpoint. Conversely, a refined thesis might assert: “Through the juxtaposition of statistical evidence and anecdotal narratives, the author constructs a persuasive argument in favor of healthcare reform.” Here, the thesis not only identifies the rhetorical strategies at play but also offers a precise interpretation of their persuasive intent.

Ultimately, the process of developing a thesis for rhetorical analysis is a dynamic interplay between observation, interpretation, and synthesis. As a diligent student of rhetoric, I am continually inspired by the transformative power of persuasive discourse. By honing the craft of thesis development, we equip ourselves with the analytical tools necessary to navigate the labyrinthine realm of rhetoric and shape discourse with precision and efficacy.

owl

Cite this page

Developing a Thesis for Rhetorical Analysis: Strategies and Examples. (2024, Apr 14). Retrieved from https://papersowl.com/examples/developing-a-thesis-for-rhetorical-analysis-strategies-and-examples/

"Developing a Thesis for Rhetorical Analysis: Strategies and Examples." PapersOwl.com , 14 Apr 2024, https://papersowl.com/examples/developing-a-thesis-for-rhetorical-analysis-strategies-and-examples/

PapersOwl.com. (2024). Developing a Thesis for Rhetorical Analysis: Strategies and Examples . [Online]. Available at: https://papersowl.com/examples/developing-a-thesis-for-rhetorical-analysis-strategies-and-examples/ [Accessed: 17 Apr. 2024]

"Developing a Thesis for Rhetorical Analysis: Strategies and Examples." PapersOwl.com, Apr 14, 2024. Accessed April 17, 2024. https://papersowl.com/examples/developing-a-thesis-for-rhetorical-analysis-strategies-and-examples/

"Developing a Thesis for Rhetorical Analysis: Strategies and Examples," PapersOwl.com , 14-Apr-2024. [Online]. Available: https://papersowl.com/examples/developing-a-thesis-for-rhetorical-analysis-strategies-and-examples/. [Accessed: 17-Apr-2024]

PapersOwl.com. (2024). Developing a Thesis for Rhetorical Analysis: Strategies and Examples . [Online]. Available at: https://papersowl.com/examples/developing-a-thesis-for-rhetorical-analysis-strategies-and-examples/ [Accessed: 17-Apr-2024]

Don't let plagiarism ruin your grade

Hire a writer to get a unique paper crafted to your needs.

owl

Our writers will help you fix any mistakes and get an A+!

Please check your inbox.

You can order an original essay written according to your instructions.

Trusted by over 1 million students worldwide

1. Tell Us Your Requirements

2. Pick your perfect writer

3. Get Your Paper and Pay

Hi! I'm Amy, your personal assistant!

Don't know where to start? Give me your paper requirements and I connect you to an academic expert.

short deadlines

100% Plagiarism-Free

Certified writers

IMAGES

  1. AP: Rhetorical Analysis Portfolio

    how to write an ap lang rhetorical analysis thesis

  2. Learn How to Write a Rhetorical Analysis Essay on Trust My Paper

    how to write an ap lang rhetorical analysis thesis

  3. How to Write the AP Lang Rhetorical Essay

    how to write an ap lang rhetorical analysis thesis

  4. How to Write a Rhetorical Analysis Thesis

    how to write an ap lang rhetorical analysis thesis

  5. How to Write a Defensible Thesis for a Rhetorical Analysis Essay

    how to write an ap lang rhetorical analysis thesis

  6. How to Write a Rhetorical Analysis Introduction

    how to write an ap lang rhetorical analysis thesis

VIDEO

  1. AP Lang Rhetorical Analysis Rubric

  2. Writing Rhetorical Analysis was HARD Until I Did THIS!

  3. Refining your Rhetorical Analysis thesis statement

  4. Rhetorical Analysis

  5. AP Lang Rhetorical Analysis Essay Review and Practice

  6. 2023 AP English Language Rhetorical Analysis Prompt Walk Through

COMMENTS

  1. How to Write the AP Lang Rhetorical Analysis Essay (With Example)

    Her story "The Astronaut" won the 2018 Shirley Jackson Award for short fiction and received a "Distinguished Stories" mention in the 2019 Best American Short Stories anthology. How to write the AP Lang rhetorical analysis essay. We look at a AP lang rhetorical analysis essay example and explore do's and don'ts.

  2. Thesis Statement Formula for AP English Rhetorical Analysis Essays

    Let's look at an example of an excellent AP rhetorical analysis thesis statement: In her indignantly critical and cleverly crafted speech given to the National Association for Women's Suffrage, Florence Kelley clearly articulates and emotionally persuades her audience through the use of parallelism and inclusive language to advocate for changes to child labor laws.

  3. How to Write the AP Lang Rhetorical Essay

    Tips for Writing the AP Lang Rhetorical Essay. 1. Outline Your Essay Before Writing. One of the most important parts of the AP Lang essays is structuring your essay so that it makes sense to the reader. This is just as important as having good content. For this essay in particular, you'll want to read the passage first and write a brief ...

  4. AP Lang

    for AP Lang: 12 minutes: Read the text and plan out your essay. (TOBI) 6 minutes: Write your introduction paragraph. 18 minutes: Write 2-3 body paragraphs. 2 minutes: Write a quick conclusion. 2 minutes: Proofread and revise your essay. đŸŽ„ Watch: AP Language - Rhetorical Analysis Organization and Timing.

  5. AP Lang

    AP Lang Rhetorical Analysis Essay Practice. Rhetorical Analysis practice is one of the most important ways to prepare for the exam! Review student writing practice samples and corresponding feedback from TA Brandon Wu! While you don't need to memorize every rhetorical device for the exam, you should take some time to familiarize yourself with them.

  6. Crafting a solid thesis for AP Lang rhetorical analysis

    Here are some tips to help you create a clear and effective thesis for your AP Lang rhetorical analysis essay: 1. Identify the author's main argument: Begin by understanding the author's main point or claim. This should become the basis for your thesis, as you'll be discussing how the author constructs this argument using rhetorical strategies. 2.

  7. PDF How to Write a RHETORICAL ANALYSIS ESSAY Step 1: Full Comprehension of

    Step 2: MAD TO WRITE! Follow this process to prepare for any timed rhetorical analysis essay. Some of this is redundant, but this portion has more to do with the actual process of writing an essay, whereas the previous questions are part of simply gaining full comprehension of the text. Main ideas - read to determine what points the speaker makes

  8. How to Write a Rhetorical Analysis

    A rhetorical analysis is a type of essay that looks at a text in terms of rhetoric. This means it is less concerned with what the author is saying than with how they say it: their goals, techniques, and appeals to the audience. A rhetorical analysis is structured similarly to other essays: an introduction presenting the thesis, a body analyzing ...

  9. How to Ace the AP Language Rhetorical Analysis Essay

    In this video, I'll show you how to write the AP English Language rhetorical analysis essay (Q2) step by step using the actual 2017 prompt. Watch me annotate...

  10. How to ace rhetorical analysis essays for AP Language?

    Hi there! Rhetorical analysis essays can indeed be challenging, but with the right mindset and a solid strategy, you can excel at them. Here are some tips and guidance to help you improve your essay writing skills: 1. Understand the rhetorical triangle: When analyzing a text, keep the rhetorical triangle in mind (i.e., the relationship between the author, audience, and purpose).

  11. AP Lang Rhetorical Analysis Essay

    The rhetorical analysis essay can seem a bit challenging at first, but with practice and a solid strategy, you can ace it! Here are some tips for tackling this essay: Start by reading the text carefully and taking notes on the author's use of rhetorical strategies, such as ethos, pathos, and logos. Jot down specific examples and make note of ...

  12. 3 APÂź English Language Rhetorical Essay Strategies

    The APÂź English Language rhetorical essay can be nightmare inducing for some APÂź students, but there is no need for fear. In this exam review we will lay out helpful strategies to get you through the rhetorical essays in no time. Rhetorical Strategy #1: Dissecting the Prompt. The first rhetorical essay strategy is to dissect the prompt.

  13. How to Write a Rhetorical Analysis Thesis

    One of the first steps of writing a rhetorical analysis essay is knowing how to write a rhetorical analysis thesis. Rhetorical analysis thesis statements can seem intimidating, but they do not have to be. While the thesis is a small portion of an essay, it carries significant weight and impact, especially on the APÂź Lang exam. For example, on ...

  14. PDF HOW TO WRITE: AP Rhetorical Analysis Paragraphs and Essays

    When writing an analysis, it is crucial that you work chronologically through the text. This means that you start at the beginning of the text and work your way through it by discussing what the writer is saying and the effectiveness of the strategies he/she is using at the beginning, middle, and end of the text.

  15. AP Lang Rhetorical Analysis: A Clear, Effective Model

    This video offers a clear, effective model for how to approach a rhetorical analysis on the AP Language and Composition Exam. I hope that it helps. Note: th...

  16. AP Lang Rhetorical Analysis Essay: Tips & Rubric

    The rhetorical analysis essay in the AP Lang exam involves students having to discuss how the authors' contribution to the passage gives a theme or meaning. A synthesis essay involves students creating arguments on the passage or piece of information delivered to them. An argumentative essay requires students to pick a side 'for or against ...

  17. Rhetorical Analysis Thesis Statement Examples

    This video explains how to write a defensible thesis statement for AP Lang Q2. This video also contains rhetorical analysis thesis statement examples.Check o...

  18. How to master the AP English Language and Composition rhetorical

    5. Use precise, concise language: Clearly articulate your analysis in a direct, concise manner. Use specific language and active verbs when analyzing the author's effectiveness. 6. Practice with past prompts: To get better at rhetorical analysis essays, practice by reviewing past AP exam prompts and analyzing various texts.

  19. AP English Language and Composition: Sample Rhetorical Analysis and

    AP English Language and Composition: Sample Argument Question. The following paragraph is adapted from Mirror for Man, a book written by anthropologist Clyde Kluckhorn in the middle of the twentieth century. Read the passage carefully. Then, write an essay that examines the extent to which the author's characterization of the United States ...

  20. AP Lang

    Question 2: The Rhetorical Analysis FRQ. The AP Lang exam's free-response section is a writing section comprised of three essays within 2 hours and 15 minutes. The second essay is on Rhetorical Analysis, in which you look at a variety of nonfiction sources - newspaper articles, essays, letters, et cetera - and make connections between the ...

  21. PDF HOW TO WRITE: AP Rhetorical Analysis Paragraphs and Essays

    D:\My Documents\Orlando Teacher docs\AP LANG and COMP\2 Close Reading The Art and Craft of Analysis Strong vs. Weak Verbs To help you move away from summary and toward ANALYSIS, you need to begin to incorporate strong verbs into your writing when discussing the writer's rhetorical choices. Below is a list of verbs that are considered

  22. How to Write the AP Lang Argument Essay + Examples

    Having a well-organized essay is crucial for success. 2. Pick one side of the argument, but acknowledge the other side. When you write the essay, it's best if you pick one side of the debate and stick with it for the entire essay. All your evidence should be in support of that one side.

  23. How to Write the AP Lang Synthesis Essay + Example

    Step 5: Write your Essay. Use the remaining 30-35 minutes to write your essay. This should be relatively easy if you took the time to mark up the sources and have a detailed outline. Remember to add special consideration and emphasis to the commentary sections of the supporting arguments outlined in your thesis.

  24. Developing a Thesis for Rhetorical Analysis: Strategies and Examples

    Central to this endeavor is the crafting of a compelling thesis for rhetorical analysis. In this essay, I aim to dissect the strategies and exemplify the process of developing such a thesis, drawing from the rich tapestry of rhetorical discourse. At its core, a rhetorical analysis thesis serves as the fulcrum upon which the entire analysis pivots.